K
Khách

Hãy nhập câu hỏi của bạn vào đây, nếu là tài khoản VIP, bạn sẽ được ưu tiên trả lời.

9 tháng 8 2020

áp dụng bđt Min-cốp-xki ta có \(\sqrt{x^2+xy+y^2}+\sqrt{x^2+xz+z^2}=\sqrt{\left(x^2+xy+\frac{y^2}{4}\right)+\frac{3y^2}{4}}+\sqrt{\left(x^2+xz+\frac{z^2}{4}\right)+\frac{3z^2}{4}}\)\(=\sqrt{\left(x+\frac{y}{2}\right)^2+\left(\frac{\sqrt{3}y}{2}\right)^2}+\sqrt{\left(-x-\frac{z}{2}\right)^2+\left(\frac{\sqrt{3}z}{2}\right)^2}\)\(\ge\sqrt{\left(x+\frac{y}{2}-x-\frac{z}{2}\right)^2+\left(\frac{\sqrt{3}y}{2}+\frac{\sqrt{3}z}{2}\right)^2}=\sqrt{\frac{y^2}{4}-\frac{yz}{2}+\frac{z^2}{4}+\frac{3y^2}{4}+\frac{3yz}{2}+\frac{3z^2}{4}}\)

\(=\sqrt{y^2+yz+z^2}\)

7 tháng 3 2021

Dễ dàng chứng minh được:

\(\frac{a^2}{x}+\frac{b^2}{y}+\frac{c^2}{z}\ge\frac{\left(a+b+c\right)^2}{x+y+z}\) với \(a,b,c>0\)(1)

Dấu bằng xảy ra \(\Leftrightarrow\frac{a}{x}=\frac{b}{y}=\frac{c}{z}\)

Theo đề bài, vì x, y, z > 0 nên áp dụng (1), ta có:

\(\frac{x^2}{x+\sqrt{yz}}+\frac{y^2}{y+\sqrt{zx}}+\frac{z^2}{z+\sqrt{xy}}\ge\)\(\frac{\left(x+y+z\right)^2}{x+y+z+\sqrt{xy}+\sqrt{yz}+\sqrt{zx}}\)(2)

Vì x y, z > 0 nên áp dụng bất đẳng thức Cô-si cho 2 số dương, ta được:

\(x+y\ge2\sqrt{xy}\)(3)

Chứng mih tương tự, ta được;

\(y+z\ge2\sqrt{yz}\)(4);

\(z+x\ge2\sqrt{zx}\)(5)

Từ (3), (4), (5), ta được:

\(2\left(x+y+z\right)\ge2\left(\sqrt{xy}+\sqrt{yz}+\sqrt{zx}\right)\)

\(\Leftrightarrow x+y+z\ge\sqrt{xy}+\sqrt{yz}+\sqrt{zx}\)

\(\Leftrightarrow2\left(x+y+z\right)\ge x+y+z+\sqrt{xy}+\sqrt{yz}+\sqrt{zx}\)

\(\Leftrightarrow\frac{1}{x+y+z+\sqrt{xy}+\sqrt{yz}+\sqrt{zx}}\ge\)\(\frac{1}{2\left(x+y+z\right)}\)

\(\Leftrightarrow\frac{\left(x+y+z\right)^2}{x+y+z+\sqrt{xy}+\sqrt{yz}+\sqrt{zx}}\ge\frac{x+y+z}{2}\)

7 tháng 3 2021

Mà theo đề bài, \(x+y+z\ge3\) nên:

\(\frac{x+y+z}{2}\ge\frac{3}{2}\)

Suy ra \(\frac{\left(x+y+z\right)^2}{x+y+z+\sqrt{xy}+\sqrt{yz}+\sqrt{zx}}\ge\frac{3}{2}\left(6\right)\)

Từ (2) và (6), ta được:

\(\frac{x^2}{x+\sqrt{yz}}+\frac{y^2}{y+\sqrt{zx}}+\frac{z^2}{z+\sqrt{xy}}\ge\frac{3}{2}\)(điều phải chứng minh)

Dấu bằng xảy ra

\(\Leftrightarrow\hept{\begin{cases}x=y=z\\x+y+z=3\end{cases}\Leftrightarrow x=y=z=1}\)

Vậy nếu x, y, z > 0 và \(x+y+z\ge3\)thì \(\frac{x^2}{x+\sqrt{yz}}+\frac{y^2}{y+\sqrt{zx}}+\frac{z^2}{z+\sqrt{xy}}\ge\frac{3}{2}\)

AH
Akai Haruma
Giáo viên
26 tháng 12 2017

Lời giải:

Áp dụng BĐT Cauchy-Schwarz ta có:

\(\text{VT}=\frac{x}{\sqrt[3]{yz}}+\frac{y}{\sqrt[3]{xz}}+\frac{z}{\sqrt[3]{xy}}=\frac{x^2}{\sqrt[3]{x^3yz}}+\frac{y^2}{\sqrt[3]{y^3xz}}+\frac{z^2}{\sqrt[3]{z^3xy}}\)

\(\geq \frac{(x+y+z)^2}{\sqrt[3]{x^3yz}+\sqrt[3]{y^3xz}+\sqrt[3]{z^3xy}}\) (1)

Áp dụng BĐT Am-Gm:

\(\sqrt[3]{x^3yz}\leq \frac{x^2+xyz+1}{3}; \sqrt[3]{y^3xz}\leq \frac{y^2+xyz+1}{3}; \sqrt[3]{z^3xy}\leq \frac{z^2+xyz+1}{3}\)

\(\Rightarrow \sqrt[3]{x^3yz}+\sqrt[3]{y^3xz}+\sqrt[3]{z^3xy}\leq \frac{x^2+y^2+z^2+3xyz+3}{3}=2+xyz\)

Theo BĐT AM-GM:

\(x^2+y^2+z^2\geq 3\sqrt[3]{x^2y^2z^2}\Leftrightarrow 3\sqrt[3]{x^2y^2z^2}\leq 3\Leftrightarrow xyz\leq 1\)

Do đó: \(\sqrt[3]{x^3yz}+\sqrt[3]{y^3xz}+\sqrt[3]{z^3xy}\leq 3\) (2)

Từ (1),(2) và sử dụng hệ quả \(x^2+y^2+z^2\geq xy+yz+xz\) :

\(\Rightarrow \text{VT}\geq \frac{(x+y+z)^2}{3}=\frac{x^2+y^2+z^2+2(xy+yz+xz)}{3}\geq \frac{3(xy+yz+xz)}{3}=xy+yz+xz\)

Ta có đpcm

Dấu bằng xảy ra khi \(x=y=z=1\)

27 tháng 12 2017

Áp dụng BĐT AM-GM ta có:

\(VT\ge\dfrac{x}{\dfrac{y+z+1}{3}}+\dfrac{y}{\dfrac{x+z+1}{3}}+\dfrac{z}{\dfrac{x+y+1}{3}}\)

Cần chứng minh \(\dfrac{9x}{y+z+1}+\dfrac{9y}{x+z+1}+\dfrac{9z}{x+y+1}\ge3\left(xy+yz+xz\right)\)

Cauchy-Schwarz: \(VT=\dfrac{9x^2}{xy+xz+x}+\dfrac{9y^2}{xy+yz+y}+\dfrac{9z^2}{xz+yz+z}\)

\(\ge\dfrac{9\left(x+y+z\right)^2}{2\left(xy+yz+xz\right)+x+y+z}\ge\left(x+y+z\right)^2\)

BĐT cuối đúng vì dễ thấy: \(\left(x+y+z\right)^2\ge3\left(xy+yz+xz\right)\)

1 tháng 1 2020

Áp dụng BĐT Cauchy - Schwarz ta có :

\(VT=\frac{x}{\sqrt[3]{yz}}+\frac{y}{\sqrt[3]{xz}}+\frac{z}{\sqrt[3]{xy}}=\frac{x^2}{\sqrt[3]{x^3yz}}+\frac{y^2}{\sqrt[3]{y^3xz}}+\frac{z^2}{\sqrt[3]{z^3xy}}\)

\(\ge\frac{\left(x+y+z\right)^2}{\sqrt[3]{x^3yz}+\sqrt[3]{y^3xz}+\sqrt[3]{z^3xy}}\left(1\right)\)

Áp dụng BĐT : AM - GM :

\(\sqrt[3]{x^3yz}\le\frac{x^2+xyz+1}{3};\sqrt[3]{y^3xz}\le\frac{y^2+xyz+1}{3};\sqrt[3]{z^3xy}\le\frac{z^2+xyz+1}{3}\)

\(\Rightarrow\sqrt[3]{x^3yz}+\sqrt[3]{y^3xz}+\sqrt[3]{z^3xy}\le\frac{x^2+y^2+z^2+3xyz+3}{3}=2+xyz\)

Theo BĐT AM - GM :

\(x^2+y^2+z^2\ge3\sqrt[3]{x^2y^2z^2}\Leftrightarrow3\sqrt[3]{x^2y^2z^2}\le3\Leftrightarrow xyz\le1\)

Do đó : \(\sqrt[3]{x^3yz}+\sqrt[3]{y^3xz}+\sqrt[3]{z^3xy}\le3\left(2\right)\)

Tư (1) , (2) và sử dụng hệ quả :
\(x^2+y^2+z^2\ge xy+yz+zx:\)

\(\Rightarrow VT\ge\frac{\left(x+y+z\right)^2}{3}=\frac{x^2+y^2+z^2+2\left(xy+yz+xz\right)}{3}\ge\frac{3\left(xy+yz+xz\right)}{3}\)\(=xy+yz+xz\)

Ta có đpcm 

Dấu " = " xảy ra khi \(x=y=z=1\)

Chúc bạn học tốt !!!

26 tháng 2 2018

\(VT=\frac{x}{\sqrt[3]{yz}}+\frac{y}{\sqrt[3]{xz}}+\frac{z}{\sqrt[3]{xy}}\)

\(\ge\frac{3x}{y+z+1}+\frac{3y}{x+z+1}+\frac{3z}{x+y+1}\)

\(=\frac{3x^2}{xy+xz+x}+\frac{3y^2}{xy+yz+y}+\frac{3z^2}{xz+yz+z}\)

\(\ge\frac{3\left(x+y+z\right)^2}{2\left(xy+yz+xz\right)+x+y+z}\)

\(\ge\frac{3\left(x+y+z\right)^2}{2\left(xy+yz+xz\right)+x^2+y^2+z^2}\)

\(\ge\frac{3\left(x+y+z\right)^2}{\left(x+y+z\right)^2}=3=x^2+y^2+z^2\ge xy+yz+xz=VP\)

Dấu "=" <=> x=y=z=1

12 tháng 3 2017

solution:

ta có: \(3=x^2+y^2+z^2\ge3\sqrt[3]{x^2y^2z^2}\Leftrightarrow xyz\le1\)(theo BĐT cauchy cho 3 số )

\(\Rightarrow xy\le\dfrac{1}{z};yz\le\dfrac{1}{x};xz\le\dfrac{1}{y}\)

\(\Rightarrow\dfrac{x}{\sqrt[3]{yz}}\ge\dfrac{x}{\dfrac{1}{\sqrt[3]{x}}}=x\sqrt[3]{x}=\sqrt[3]{x^4}\)

tương tự ta có:\(\dfrac{y}{\sqrt[3]{xz}}\ge\sqrt[3]{y^4};\dfrac{z}{\sqrt[3]{xy}}\ge\sqrt[3]{z^4}\)

cả 2 vế các BĐT đều dương,cộng vế với vế:

\(S=\dfrac{x}{\sqrt[3]{yz}}+\dfrac{y}{\sqrt[3]{xz}}+\dfrac{z}{\sqrt[3]{xy}}\ge\sqrt[3]{x^4}+\sqrt[3]{y^4}+\sqrt[3]{z^4}\)

Áp dụng BĐT bunyakovsky ta có:

\(\left(\sqrt[3]{x^4}+\sqrt[3]{y^4}+\sqrt[3]{z^4}\right)\left(x^2+y^2+z^2\right)\ge\left(\sqrt[3]{x^8}+\sqrt[3]{y^8}+\sqrt[3]{z^8}\right)^2=\left(x^2+y^2+z^2\right)^2\)

\(\Rightarrow S\ge x^2+y^2+z^2\)

đến đây ta lại có BĐT quen thuộc: \(x^2+y^2+z^2\ge xy+yz+xz\)

\(\Rightarrow S\ge xy+yz+xz\left(đpcm\right)\)

dấu = xảy ra khi và chỉ khi x=y=z mà x2+y2+z2=3 => x=y=z=1

*cách khác : Áp dụng BĐT cauchy - schwarz(bunyakovsky):

\(S=\dfrac{x}{\sqrt[3]{yz}}+\dfrac{y}{\sqrt[3]{xz}}+\dfrac{z}{\sqrt[3]{xy}}=\dfrac{x^4}{x^3.\dfrac{1}{\sqrt[3]{x}}}+\dfrac{y^4}{y^3.\dfrac{1}{\sqrt[3]{y}}}+\dfrac{z^4}{z^3.\dfrac{1}{\sqrt[3]{z}}}\)

\(S\ge\dfrac{\left(x^2+y^2+z^2\right)^2}{x^2+y^2+z^2}=x^2+y^2+z^2\ge xy+yz+xz\)

13 tháng 3 2017

cái cách 2 là svac mà nhỉ

5 tháng 2 2020

Áp dụng BĐT Cô-si dạng Engel,ta có :

\(\frac{x^2}{x+\sqrt{yz}}+\frac{y^2}{y+\sqrt{xz}}+\frac{z^2}{z+\sqrt{xy}}\ge\frac{\left(x+y+z\right)^2}{x+y+z+\sqrt{xy}+\sqrt{yz}+\sqrt{xz}}\)

Mà \(\sqrt{xy}+\sqrt{yz}+\sqrt{xz}\le x+y+z\)

\(\Rightarrow\)\(\frac{\left(x+y+z\right)^2}{x+y+z+\sqrt{xy}+\sqrt{yz}+\sqrt{xz}}\ge\frac{\left(x+y+z\right)^2}{2\left(x+y+z\right)}=\frac{x+y+z}{2}\ge\frac{3}{2}\)

Dấu "=" xảy ra khi x = y = z = \(\frac{3}{2}\)

5 tháng 2 2020

nhầm sửa x = y = z = 1 nha

9 tháng 2 2017

Áp dụng bất đẳng thức cộng mẫu số

\(\Rightarrow\frac{x^2}{x+\sqrt{yz}}+\frac{y^2}{y+\sqrt{xz}}+\frac{z^2}{z+\sqrt{xy}}\ge\frac{\left(x+y+z\right)^2}{x+y+z+\sqrt{yz}+\sqrt{xz}+\sqrt{xy}}\)

Xét \(\frac{\left(x+y+z\right)^2}{x+y+z+\sqrt{yz}+\sqrt{xz}+\sqrt{xy}}\)

Áp dụng bất đẳng thức Cauchy cho 2 bộ số thực không âm

\(\Rightarrow\left\{\begin{matrix}\sqrt{yz}\le\frac{y+z}{2}\\\sqrt{xz}\le\frac{x+z}{2}\\\sqrt{xy}\le\frac{x+y}{2}\end{matrix}\right.\)

\(\Rightarrow\sqrt{yz}+\sqrt{xz}+\sqrt{xy}\le\frac{y+z}{2}+\frac{x+z}{2}+\frac{x+y}{2}\)

\(\Rightarrow\sqrt{yz}+\sqrt{xz}+\sqrt{xy}\le\frac{2\left(x+y+z\right)}{2}\)

\(\Rightarrow\sqrt{yz}+\sqrt{xz}+\sqrt{xy}\le x+y+z\)

\(\Rightarrow x+y+z+\sqrt{xy}+\sqrt{yz}+\sqrt{xz}\le2\left(x+y+z\right)\)

\(\Rightarrow\frac{\left(x+y+z\right)^2}{x+y+z+\sqrt{xy}+\sqrt{xz}+\sqrt{yz}}\ge\frac{\left(x+y+z\right)^2}{2\left(x+y+z\right)}=\frac{x+y+z}{2}\)

Ta có: \(x+y+z\ge3\)

\(\Rightarrow\frac{x+y+z}{2}\ge\frac{3}{2}\)

\(\Rightarrow\frac{\left(x+y+z\right)^2}{x+y+z+\sqrt{xy}+\sqrt{xz}+\sqrt{yz}}\ge\frac{3}{2}\)

\(\frac{x^2}{x+\sqrt{yz}}+\frac{y^2}{y+\sqrt{xz}}+\frac{z^2}{z+\sqrt{xy}}\ge\frac{\left(x+y+z\right)^2}{x+y+z+\sqrt{yz}+\sqrt{xz}+\sqrt{xy}}\)

\(\Rightarrow\frac{x^2}{x+\sqrt{yz}}+\frac{y^2}{y+\sqrt{xz}}+\frac{z^2}{z+\sqrt{xy}}\ge\frac{3}{2}\) ( đpcm )